Đến nội dung

vda2000 nội dung

Có 295 mục bởi vda2000 (Tìm giới hạn từ 19-04-2020)



Sắp theo                Sắp xếp  

#693799 Tuần $4$ tháng $9/2017$: $AP$ đi qua điểm cố định

Đã gửi bởi vda2000 on 27-09-2017 - 14:39 trong Chuyên mục Mỗi tuần một bài toán Hình học

Lời giải bài 1 của em: 

Như trên hình, em chỉ xét những điểm liên quan đến $N$.

Gọi $S, D$ lần lượt là điểm chính giữa cung $ABC, BC$ của $(O)$. Gọi $V$ là trung điểm $AC$ và $SE$ cắt $DF$ tại $U$. Gọi $AI$ cắt $BC$ tại $G$. Gọi $J$ là trung điểm của $AI$

Bằng cách định nghĩa ta dễ có: $N\in AS$

abc.jpg

Nhận xét 1: $NQ$ vuông góc $AC$.

Chứng minh: Điều này tương đương với $NQ//SV$ hay theo định lý Ta-lét, ta cần chứng minh:

$\frac{AN}{AS}=\frac{AQ}{AV}$. Do: $\frac{AQ}{AV}=\frac{AI}{AG}$ dựa trên $JQ//AC$ và $J,V$ lần lượt là trung điểm của $AI,AC$.

Do đó, ta cần chứng minh: $\frac{AN}{AS}=\frac{AI}{AG}\Leftrightarrow\frac{NA}{NS}=\frac{IA}{IG}$, luôn đúng.

Do

 

Nhận xét 2: $AP$ là đường đối trung trong tam giác $\Delta RAQ$.

Ta chứng minh điều này dựa vào định lý Ceva-sin.

Ta tính được: $\frac{\text{sin}\widehat{LQR}}{\text{sin}\widehat{LQA}}=\text{cos}(\frac{A}{2}).\frac{\text{cos}(\frac{C}{2})}{\text{sin}(\frac{B}{2})}$ $(1)$

Thiết lập, tương tự, ta có: $\frac{\text{sin}\widehat{KRA}}{\text{sin}\widehat{KRQ}}=1:\text{cos}(\frac{A}{2}).\frac{\text{sin}(\frac{C}{2})}{\text{cos}(\frac{B}{2})}$.

Nhân lại, sử dụng định lý sin và định lý Ceva-sin, ta có ngay:

$\frac{\text{sin}\widehat{RAP}}{\text{sin}\widehat{QAP}}=\frac{\text{sin}\widehat{AQR}}{\text{sin}\widehat{ARQ}}$, đủ để suy ra nhận xét.

 

Do đó, ta chứng minh $(1)$.

 

Cuối cùng, vì $AP$ là đường đối trung tam giác $RAQ$ nên cũng là đối trung tam giác $ABC$ nên đi qua điểm cố định là giao hai tiếp tuyến tại $B,C$ của $(O)$. $\blacksquare$




#693455 Đề thi chọn học sinh giỏi THPT Khoa Học Tự Nhiên 2017-2018

Đã gửi bởi vda2000 on 21-09-2017 - 07:31 trong Thi HSG cấp Tỉnh, Thành phố. Olympic 30-4. Đề thi và kiểm tra đội tuyển các cấp.

Bài 2. 

Dễ thấy $P(x)=(x^3-3)Q(x)+2017$

Đặt $Q(x)=a_{n} x^n+a_{n-1} x^{n-1}+...+a_0$ 

$P(x)=a_{n} x^{n+3}+a_{n-1} x^{n+2} +a_{n-2} x^{n+1} +( a_{n-3}-3a_{n} ) x^n+(a_{n-4}-3a_{n-1}) x^{n-1}+...+(a_{0}-3a_{3})x^3-3a_{2} x^2 -3a_{1} x +2017 - 3a_{0}$

Do $P(x)$ có hệ số không âm nên ta phải có hệ
$$\left\{\begin{matrix} a_{n},a_{n-1},a_{n-2} \geq 0\\ a_{n-3} \geq 3a_{n}\geq 0\\ ...\\ a_{0} \geq 3 a_{3} \geq 0\\ a_1 ,a_2 \leq 0\\ a_0 \leq \dfrac{2017}{3}\\ \end{matrix}\right.$$

Cho ta các nghiệm nguyên không âm $a_{n}=a_{n-1}=...=a_{1}=0$ hay $Q(x)=a_0=c \leq 672$ là hàm hằng. 

$P(1)=c+2017-3c=2017-2c \geq 673$

Dấu "=" xảy ra khi $P(x)=672 x^3 +1$

$P(x)=1+2x^6+2x^9+2x^{15}+2x^{18}$ $P(\sqrt[3]{3})=2017, P(1)=9<674$




#690683 Tuần 3 tháng 8/2017: $PQ$ chia đôi $CD$

Đã gửi bởi vda2000 on 16-08-2017 - 17:27 trong Chuyên mục Mỗi tuần một bài toán Hình học

Một cách tiếp cận khác: Xét phép nghịch đảo cực $R$, phương tích bất kỳ (bản chất chắc nó cũng không quá khác so với 2 lời giải trên), khi đó, ta có bài toán như hình vẽ:

 

Cho $4$ điểm $B,C,E,F$ thuộc một đường tròn $(O)$, $BE,CF$ cắt nhau tại $R$. $(RFB),(REC)$ cắt nhau tại $A$ (Chú ý, từ điều kiện này và tính chất $3$ trục đẳng phương, suy ra: $BF,CE$ đồng quy tại $K$). $M,N$ thuộc $(O)$ sao cho $2$ tiếp tuyến tại $2$ điểm này với $(O)$ cùng song song với $RK$.

Chứng minh rằng: $(RFM), (REN)$ cắt nhau trên $RK$ (Điều kiện này cũng tương đương với $MF,NE$ cắt nhau tại $S$ thì $S$ thuộc $KR$).

Tóm lại, ta phải chỉ ra: $S$ thuộc $KR$.

Vì $2$ tiếp tuyến tại $M,N$ với $(O)$ song song với $RK$, nên theo định lý Brokard, suy ra: $EF,MN,BC$ đồng quy tại $T$.

Gọi $FN,ME$ cắt nhau tại $J$ thì $J$ thuộc đường thẳng cực của $T$, nên: $K,J,R$ thẳng hàng.

Cuối cùng, lại theo định lý Brokard một lần nữa, vì: $OT\bot SJ, KR$ nên $S$ thuộc $KR$. $\blacksquare$

 

 




#679779 BMO 2017

Đã gửi bởi vda2000 on 07-05-2017 - 09:18 trong Thi HSG Quốc gia và Quốc tế

Bài 3:

Theo em biết thì nước ngoài hiểu $\mathbb{N}={1,2,...}$

Giả sử một hàm $f$ thỏa mãn đề bài.

Ta có: $n+f(m)|f(n)+nf(m)$ nên: $n+f(m)|f(n)-n^2$ $(1)$ và: $n+f(m)|f(n)-[f(m)]^2$ $(2)$

Giả sử rằng $f(m)$ chỉ nhận hữu hạn giá trị trên $\mathbb{N}$

Khi đó, tồn tại tập $A={n_1,n_2,...}$ có vô hạn phần tử, được sắp thứ tự tăng dần sao cho: $f(n_1)=f(n_2)=...=a$

Thay $m=n_1, n=n_i$, suy ra:

$n_i+a|a-a^2$. Theo cách định nghĩa của ta, cho $i\rightarrow +\infty$ thì $n_i\rightarrow +\infty$, suy ra:

$a-a^2=0$, vì xét hàm trên $\mathbb{N}$ suy ra: $a=1$

Cố định $m_0$ trong $(2)$, thay $n=n_i$, suy ra:

$n_i+f(m_0)|1-[f(m_0)]^2$

Một lần nữa, cho $i\rightarrow +\infty$, $n_i\rightarrow +\infty$, dẫn đến: $f(m_0)=1$, từ đó suy ra: $f(x)=1$, với mọi $x\in\mathbb{N}$

Trường hợp tập giá trị của $f(m)$ không bị chặn, làm giống bạn @Minhnksc




#667086 Đề Thi VMO năm 2017

Đã gửi bởi vda2000 on 05-01-2017 - 14:28 trong Thi HSG Quốc gia và Quốc tế

Tất cả a đều thoả

thực chất $a\geq\frac{-1}{10}$ để $u_2$ xác định nữa




#662890 Tuần 4 tháng 11/2016 : Trục đẳng phương đi qua trực tâm

Đã gửi bởi vda2000 on 24-11-2016 - 11:27 trong Chuyên mục Mỗi tuần một bài toán Hình học

Ta chứng minh hai sự kiện sau:
 
1) $EF$ là trục đẳng phương của $(O)$ và đường tròn đường kính $LH$ với $E,F$ là chân đường cao hạ từ $B,C$ xuống $AC,AB$.
PART1.jpg
Gọi $d$ là trục đẳng phương của $(O)$ và đường tròn đường kính $LH$, $R$ là giao điểm của đường tròn đường kính $AH$ với $(O)$, khác $A$ và $Z$ là chân đường vuông góc hạ từ $H$ xuống $AM$
Gọi $U,V$ lần lượt là trung điểm của: $LH,LA$, ta có một số điều sau suy ra từ đường trung bình trong tam giác:
$UV//AH//OM$ và $UV=OM=\frac{AH}{2}$, suy ra $UVOM$ là hình bình hành, suy ra $U,A,O$ thằng hàng.
Xét $3$ đường tròn sau: đường tròn đường kính $AH$, đường tròn đường kính $LH$ và đường tròn $(O)$, suy ra:
$d$, $AR$, $HZ$ đồng quy.
Theo một kết quả khá quen thuộc, ta có: $AR,HZ,EF,BC$ cùng đi qua điểm $S$ nên $S$ thuộc $d$.
Mà: $d\bot UO\equiv AO$, $EF\bot AO$ nên ta có: $d\equiv EF$, ta chứng minh xong sự kiện thứ nhất.
 
2) Ta sẽ chứng minh kết quả của bài toán.
Gọi $J,M$ lần lượt là trung điểm của $AH,BC$ $ON\bot PH$, $H'$ đối xứng với $H$ qua $OM$
Bằng cách lược bỏ đi một số điểm không cần thiết như $L,K,...$, ta có hình sau:
PART2.jpg
Điều phải chứng minh tương đương với:
$HK.HO=HJ.HD$

Spoiler

Hay điều phải chứng minh tương đương với: $\frac{SB}{BE}=\frac{HB}{HH'}$, nhưng điều này luôn đúng vì:
$\widehat{SBE}=\widehat{BHH'}$ và $\widehat{SEB}=\widehat{HCB}=\widehat{BH'H}$.
Vậy ta có điều phải chứng minh




#655661 Đề thi chọn đội tuyển quốc gia THPT chuyên KHTN - ĐHQG Hà Nội vòng 2 năm 2016

Đã gửi bởi vda2000 on 26-09-2016 - 20:50 trong Thi HSG cấp Tỉnh, Thành phố. Olympic 30-4. Đề thi và kiểm tra đội tuyển các cấp.

hình.jpg

Lời giải bài hình của em:

Lời giải.
a) Ta gọi $AH$ cắt $(O)$ tại điểm thứ hai là $A'. HG$ cắt $BC$ tại $ F'$
Ta sẽ chứng minh $F,F',A'$ thẳng hàng.
Thật vậy: $∠F'HA'=∠F'A'H=∠EAH=∠FA'H$ ( vì  $HP// AE$ và $AEFA'$ là hình thang cân do $EF//AA'$) , suy ra $F,F',A'$ thẳng hàng.
Khi  đó, ta có: $∠FGF'=∠QAE=∠FA'P$ (dựa vào tính song song của gt và $AQPA'$ là hình thang cân) , suy ra:$ PFGA' $nội tiếp, nên theo tính chất phương tích:
$F'P.F'G=F'F.F'A'=F'B.F'C$, suy ra: $BPCG$ nội tiếp (đpcm)
 
b) Gọi $A'F$ cắt trung trực của $BC$ tại $S$.
Theo định lý Reim: $QS//AA'; FRA'A$ nội tiếp nên: $FRSQ$ nội tiếp.
Phần còn lại em dùng tính toán và mọi người chắc cũng không thích xem nó.
Tính toán
 

 




#648921 $AK$ luôn đi qua một điểm cố định

Đã gửi bởi vda2000 on 10-08-2016 - 18:31 trong Hình học

Cho hai đường tròn $(O_1)$, $(O_2)$ cắt nhau tại $B,C$. $M$ là trung điểm $BC$. Gọi $A$ là điểm di chuyển trên $(O_1)$, $AB,AC$ lần lượt cắt $(O_2)$ tại điểm thứ hai là $F,E$. Gọi $P,Q$ lần lượt là hình chiếu của $M$ lên $BE,CF$.

Chứng minh rằng: $AK$ luôn đi qua một điểm cố định




#643077 Chu vi tứ giác

Đã gửi bởi vda2000 on 01-07-2016 - 10:15 trong Hình học

Chứng minh rằng chu vi của tứ giác (không nhất thiết phải lồi) bị chứa luôn nhỏ hơn chu vi của tứ giác lồi mà chứa tứ giác đó.




#632090 CMR: $\sum \frac{ab}{(a-b)^2}\ge \frac{-1}{4}$

Đã gửi bởi vda2000 on 09-05-2016 - 17:12 trong Bất đẳng thức và cực trị

Ta thấy rằng, nếu đặt: $(x,y,z)\rightarrow(\frac{a+b}{a-b};\frac{b+c}{b-c};\frac{c+a}{c-a})$ thì:

$(x+1)(y+1)(z+1)=(x-1)(y-1)(z-1)$ hay: $xy+yz+zx=-1$

Mặt khác: $(x+y+z)^2\geq 0$ hay:

$x^2+y^2+z^2\geq -2(xy+yz+zx)=2$

$\Leftrightarrow \sum\frac{(a+b)^2}{(a-b)^2}\geq 2$

$\Leftrightarrow\sum\frac{(a-b)^2+4ab}{(a-b)^2}\geq 2$

$\Leftrightarrow\sum\frac{ab}{(a-b)^2}\geq\frac{-1}{4}$




#631923 ĐỀ THI OLYMPIC CHUYÊN KHOA HỌC TỰ NHIÊN 2016

Đã gửi bởi vda2000 on 08-05-2016 - 12:58 trong Thi HSG cấp Tỉnh, Thành phố. Olympic 30-4. Đề thi và kiểm tra đội tuyển các cấp.

Câu c theo Brokard có: $IL\bot MZ$ với $Z$ là giao điểm của $ED$ với $PF$.

Còn theo định lý Pascal thì: $M,B,Z$ thẳng hàng nên: $MZ$ vuông góc $BC$

Suy ra $IL$ song song với $BC$ nên $I$ thuộc $LK$




#631672 Cho $\frac{1}{x^{^{2}}}+...

Đã gửi bởi vda2000 on 06-05-2016 - 22:37 trong Bất đẳng thức và cực trị

$\frac{y^2z^2}{x(y^2+z^2)}\geq\frac{3\sqrt{3}}{2}\frac{1}{x^2}$




#630647 TURKEY Team Selection Test 2016

Đã gửi bởi vda2000 on 01-05-2016 - 21:21 trong Thi HSG Quốc gia và Quốc tế

Lời giải. $AC,AB$ lần lượt cắt $\odot (BPC)$ tại $P,Q$.

attachicon.gifPost 93.png

Hình vẽ bài toán

Ta có: $\angle BQL=\angle BCL=\angle BLE=\angle BAN\Rightarrow AN\parallel QL\Rightarrow \frac {AE}{EQ}=\frac {NE}{NL}$

Tương tự ta suy ra $\frac {DM}{DK}=\frac {DA}{DP}$

$\Rightarrow \frac {DK}{DM}=\frac {EL}{EN}$ khi và chỉ khi $DE\parallel PQ$.

Mặt khác do $B,C,Q,L$ đồng viên nên $\angle PQB=\angle BCA$

$\Rightarrow DE\parallel PQ$ khi và chỉ khi $BCDE$ là tứ giác nội tiếp hay $P$ là trực tâm tam giác $ABC$.

Nhận xét: Nên phát biểu là $DK=DM$ và $EL=EN$ thì bài sẽ đẹp hơn! :)

Có tí thắc mắc, đoạn này là bổ đề nào vậy nhỉ?




#616784 $\left\{\begin{matrix} \sqrt{x^2...

Đã gửi bởi vda2000 on 24-02-2016 - 22:13 trong Phương trình - hệ phương trình - bất phương trình

Giải hệ phương trình:

$\left\{\begin{matrix} \sqrt{x^2+x+1}-\sqrt{y^2-y+1}=\sqrt{x^2+xy+y^2} &\\ 4(x+1)(xy+y-1)-3x=\sqrt[3]{x^4-x^2} \end{matrix}\right.$




#598329 $\sum\frac{a}{b}+\frac{2(\s...

Đã gửi bởi vda2000 on 14-11-2015 - 21:18 trong Bất đẳng thức và cực trị

Chứng minh rằng với mọi $a,b,c>0$ thì:

$\frac{a}{b}+\frac{b}{c}+\frac{c}{a}+\frac{2(ab+bc+ca)}{a^2+b^2+c^2}\geq 5$




#597818 Chứng minh rằng: $(1+x_1)...\geq\sqrt{(n+1)^{n+1...

Đã gửi bởi vda2000 on 11-11-2015 - 12:35 trong Bất đẳng thức và cực trị

Cho $n\in\mathbb{N^*}$ và: $x_1;x_2;...;x_n>0$

Chứng minh rằng: $(1+x_1)(1+x_1+x_2)...(1+x_1+x_2+...+x_n)\geq\sqrt{(n+1)^{n+1}x_1x_2...x_n}$




#597536 Đề thi chọn đội tuyển thi VMO của khối THPT chuyên ĐHKH Huế 2016

Đã gửi bởi vda2000 on 09-11-2015 - 18:15 trong Thi HSG cấp Tỉnh, Thành phố. Olympic 30-4. Đề thi và kiểm tra đội tuyển các cấp.

Đề thi chọn đội tuyển thi VMO của khối THPT chuyên ĐHKH Huế 2016

Bài 4:

$2VT-2VP=\sum a^2(b^2+c^2)-2\sum a=\sum [a^2(3-a^2)-2a]=-\sum a(a+2)(a-1)^2\leq 0$




#596695 $xyz(x-1)(y-1)(z-1) \le 8$

Đã gửi bởi vda2000 on 03-11-2015 - 17:17 trong Bất đẳng thức - Cực trị

Cho $x,y,z \ge 0$ thỏa $xyz=x+y+z+2$. Chứng minh rằng:

$xyz(x-1)(y-1)(z-1) \le 8$

Xem ở đây.




#596504 Phép quay quanh $1$ điểm

Đã gửi bởi vda2000 on 01-11-2015 - 22:07 trong Hình học

Cho $4$ điểm: $A,B,C,D$ và $M$.

Thực hiện liên tiếp các phép quay tâm $A,B,C,D$ góc $90^{\circ}$ theo chiều ngược đồng hồ biến điểm $M$ sau $4$ lần thành chính nó.

Chứng minh rằng: $AB=CD; AB\bot CD$




#596502 $6x^3-24x^2+31x-2=6\sqrt[3]{6x^2-10x+4}$

Đã gửi bởi vda2000 on 01-11-2015 - 21:59 trong Phương trình - hệ phương trình - bất phương trình

Giải phương trình với $x\geq 1$:

$6x^3-24x^2+31x-2=6\sqrt[3]{6x^2-10x+4}$




#595791 Tìm tất cả các số thỏa mãn

Đã gửi bởi vda2000 on 28-10-2015 - 18:41 trong Tổ hợp - Xác suất và thống kê - Số phức

Có bao nhiêu số $a$ thỏa mãn:

1) $a$ có $5$ chữ số khác nhau và chữ số $1$ và $2$ cạnh nhau

2) $a$ có $7$ chữ số khác nhau và chữ số $1$ cách chữ số $2$ một chữ số.

3) $a$ có $9$ chữ số, trong đó, chữ số $1$ xuất hiện $3$ lần, các chữ số còn lại khác nhau.




#589377 Chứng minh rằng: $h=1$ hoặc $h=2^{max(1;s-u+1)}...

Đã gửi bởi vda2000 on 16-09-2015 - 20:51 trong Số học

Cho $a,b,c,n,u,v,h$ là các số nguyên dương, $b,c$ là số lẻ thỏa mãn:

$\left\{\begin{matrix} n=2^s &\\ a-1=2^u.b &\\ a^2-1=2^v.c \end{matrix}\right.$

Đặt $h=ord_n(a)$

Chứng minh rằng:

1) Nếu: $u\geq s$ thì $h=1$

2) Nếu: $u<s$ thì $h=2^{max(1;s-u+1)}$

 




#589267 Đề thi khảo sát tháng 9 năm 2015 THPT Chuyên Bắc Giang

Đã gửi bởi vda2000 on 16-09-2015 - 12:02 trong Tài liệu tham khảo khác

Sở GD& ĐT Bắc Giang                                         ĐỀ THI KHẢO SÁT THÁNG 9 - NĂM 2015

    Trường THPT                                                          MÔN: TOÁN LỚP 10 CHUYÊN

 Chuyên Bắc Giang                                                     Thời gian làm bài: 150 phút

                                                                                   Đề dành cho lớp 10 chuyên Toán.

 

Câu 4: Cho hình bình hành $ABCD$ có $\widehat{BAD}<90^{\circ}$. $O$ nằm trong tam giác $ABD$: $OC$ không vuông góc với $BD$. $(O;OC)$ cắt $BD$ tại $M,N$ ($B$ thuộc đoạn $MD$). Tiếp tuyến tại $C$ của $(O)$ cắt $AD,AB$ lần lượt tại $P,Q$

a) CMR: $M,N,P,Q$ cùng thuộc một đường tròn.

b) $CM$ cắt $QN$ tại $K$. $CN$ cắt $PM$ tại $L$. Chứng minh rằng: $IC\bot KL$

 

Thôi nói tóm tắt

a) Gọi $MN$ cắt $PQ$ tại $X$

Ta có: $XM.XN=XC^2$

$XC^2=XP.XQ$

Suy ra: $XM.XN=XP.XQ$

$\Rightarrow ...$

b) Dựa vào câu a rồi chứng minh tứ giác $MNKL$ nội tiếp




#589042 Đề thi khảo sát tháng 9 năm 2015 THPT Chuyên Bắc Giang

Đã gửi bởi vda2000 on 15-09-2015 - 11:00 trong Tài liệu tham khảo khác

Sở GD& ĐT Bắc Giang                                         ĐỀ THI KHẢO SÁT THÁNG 9 - NĂM 2015

    Trường THPT                                                          MÔN: TOÁN LỚP 10 CHUYÊN

 Chuyên Bắc Giang                                                     Thời gian làm bài: 150 phút

                                                                                   Đề dành cho lớp 10 chuyên Toán.

 

Câu 1: Giải các phương trình sau:

1) $3\sqrt{x-1}+2\sqrt{x-2}=\sqrt{x^2-3x+2}+6$

2) $x+1+\sqrt{x^2-4x+1}=3\sqrt{x}$

 

Câu 2: Giải các hệ phương trình sau:

1) $\left\{\begin{matrix} xy+x+y=x^2-2y^2 &\\ x\sqrt{2y}-y\sqrt{x-1}=2x-2y\end{matrix}\right.$

2) $\left\{\begin{matrix} 6\frac{x}{y}-2=\sqrt{3x-y}+3y &\\ 2\sqrt{3x+\sqrt{3x-y}}=6x+3y-4 \end{matrix}\right.$

 

Câu 3: Tìm tất cả các số nguyên tố $p$ sao cho: 

$3^p+4^p$ là số chính phương

 

Câu 4: Cho hình bình hành $ABCD$ có $\widehat{BAD}<90^{\circ}$. $O$ nằm trong tam giác $ABD$: $OC$ không vuông góc với $BD$. $(O;OC)$ cắt $BD$ tại $M,N$ ($B$ thuộc đoạn $MD$). Tiếp tuyến tại $C$ của $(O)$ cắt $AD,AB$ lần lượt tại $P,Q$

a) CMR: $M,N,P,Q$ cùng thuộc một đường tròn.

b) $CM$ cắt $QN$ tại $K$. $CN$ cắt $PM$ tại $L$. Chứng minh rằng: $IC\bot KL$

 

Câu 5: Cho $a,b,c>0$ thỏa mãn: $\frac{1}{a}+\frac{1}{b}+\frac{1}{c}=3$

CMR: $\frac{a}{a^4+1+2ab}+\frac{b}{b^4+1+2bc}+\frac{c}{c^4+1+2ca}\leq\frac{3}{4}$

 

 




#584737 Chứng minh rằng: $\widehat{BEC}=\widehat{DAC...

Đã gửi bởi vda2000 on 24-08-2015 - 21:18 trong Hình học phẳng

Cho tứ giác $ABCD$ và $AD=CD$, $\widehat{DAB}=\widehat{ABC}$. $M$ là trung điểm $BC$. $DM$ cắt $AB$ tại $E$. 

Chứng minh rằng: $\widehat{BEC}=\widehat{DAC}$

abc.jpg